Eigenvector with Complex Eigenvalues - What am I doing wrong?

Click For Summary
The discussion focuses on solving for eigenvectors with complex eigenvalues. A mistake was identified in the order of variables in the attempted solution, specifically swapping x1 and x2. By setting x2 to 1, the correct eigenvector was derived as (1, -1 - i), which can be transformed to (i, 1 - i) by multiplying by i. The participants emphasize the importance of correctly applying matrix reduction and the conjugate of complex numbers in the process. Overall, the conversation highlights common pitfalls when dealing with complex eigenvalues in linear algebra.
alsvt
Messages
3
Reaction score
0

Homework Statement


eigenvector.png

Homework Equations


Conjugate of a complex number
Matrix reduction

The Attempt at a Solution



My attempt is bordered. Sorry about the quality.
snapshot.jpg


So I'm not sure what I'm missing. I use the exact same method that I use for normal eigenvectors, just with complex numbers in the mix.
 
Last edited:
Physics news on Phys.org
You messed up in the last step and swapped x1 and x2. If you let x2=1, then your equation gave you x1=-1/2+1/2 i. In the vector, however, you have the two values in the other order.
 
I can't quite read all of your work, but if you subtract \lambda along the diagonal, you get:

\bmatrix<br /> 1 + i &amp; 1 \\<br /> -2 &amp; -1 + i \\<br /> \endbmatrix x = 0

The rows of this matrix are multiples (-1 + i) of each other, so you can use either row to find the eigenvector:

x_1 (1 + i) + x_2 = 0
-x_1 (1 + i) = x_2

so your vector is (1, -1 - i). This happens to not be one of your choices, but you can multiply it by i to obtain (i, 1 - i).
 
Last edited:
@vela Thanks. I wasn't really thinking it through at the end.
 
Question: A clock's minute hand has length 4 and its hour hand has length 3. What is the distance between the tips at the moment when it is increasing most rapidly?(Putnam Exam Question) Answer: Making assumption that both the hands moves at constant angular velocities, the answer is ## \sqrt{7} .## But don't you think this assumption is somewhat doubtful and wrong?

Similar threads

  • · Replies 7 ·
Replies
7
Views
2K
  • · Replies 5 ·
Replies
5
Views
2K
  • · Replies 3 ·
Replies
3
Views
2K
  • · Replies 2 ·
Replies
2
Views
2K
  • · Replies 6 ·
Replies
6
Views
2K
  • · Replies 19 ·
Replies
19
Views
4K
  • · Replies 5 ·
Replies
5
Views
14K
  • · Replies 3 ·
Replies
3
Views
1K
Replies
9
Views
2K
  • · Replies 2 ·
Replies
2
Views
2K